www.vorhilfe.de
- Förderverein -
Der Förderverein.

Gemeinnütziger Verein zur Finanzierung des Projekts Vorhilfe.de.
Hallo Gast!einloggen | registrieren ]
Startseite · Mitglieder · Impressum
Forenbaum
^ Forenbaum
Status VH e.V.
  Status Vereinsforum

Gezeigt werden alle Foren bis zur Tiefe 2

Navigation
 Startseite...
 Suchen
 Impressum
Das Projekt
Server und Internetanbindung werden durch Spenden finanziert.
Organisiert wird das Projekt von unserem Koordinatorenteam.
Hunderte Mitglieder helfen ehrenamtlich in unseren moderierten Foren.
Anbieter der Seite ist der gemeinnützige Verein "Vorhilfe.de e.V.".
Partnerseiten
Weitere Fächer:

Open Source FunktionenplotterFunkyPlot: Kostenloser und quelloffener Funktionenplotter für Linux und andere Betriebssysteme
Forum "Gruppe, Ring, Körper" - ord(gh) = ord(hg)
ord(gh) = ord(hg) < Gruppe, Ring, Körper < Algebra < Algebra+Zahlentheo. < Hochschule < Mathe < Vorhilfe
Ansicht: [ geschachtelt ] | ^ Forum "Gruppe, Ring, Körper"  | ^^ Alle Foren  | ^ Forenbaum  | Materialien

ord(gh) = ord(hg): Frage (beantwortet)
Status: (Frage) beantwortet Status 
Datum: 06:37 Mo 06.08.2012
Autor: ChopSuey

Aufgabe
Sei $ G $ eine Gruppe und $ h,g [mm] \in [/mm] G $. Zeige: $ ord(hg) = ord(gh)$


Hi Leute!

ich bräuchte Hilfe bei obiger Aufgabe.
Mein erster Gedanke war zu Zeigen, dass die zyklischen Gruppen von $ hg  $ und $ gh$ gleich sind, das war aber nicht wirklich zielführend.

Auch durch den Ansatz "sei $ [mm] \ord(gh) [/mm] = m$ also $ [mm] (gh)^m [/mm] = e $ wobei $ e $ das neutrale Element aus $ G $ bezeichnet und $ m [mm] \in \IN [/mm] $, kam ich nicht weit.

Offenbar lässt sich das Ganze über die Konjugation beweisen. Ich weiß nur noch nicht, wie. Anscheinend haben konjugierte Elemente einer Gruppe  $ G $ dieselbe Ordnung. Doch woraus folgt das? In meinem Skript steht dazu leider nichts.

Was ich mir denken kann, ist dass durch die Bijektion der Konjugationsabbildung die Gleichheit der Ordnung konjugierter Elemente gesichert wird. Bin mir allerdings nicht ganz sicher.

Ich weiß auch nicht, wie ich zeigen soll/kann, dass $ gh$ und $hg $ konjugierte sind.

Jemand einen Tipp für mich?

Vielen Dank!

Viele Grüße
ChopSuey

        
Bezug
ord(gh) = ord(hg): Antwort
Status: (Antwort) fertig Status 
Datum: 07:58 Mo 06.08.2012
Autor: Schadowmaster

moin,

Was hälst du von der Konjugation
[mm] $h(gh)h^{-1}$? [/mm]
Die Elemente sind also konjugiert.
Jetzt können wir die Frage stellen, wieso konjugierte Elemente dieselbe Ordnung haben.
Betrachte dafür zwei Untergruppen $U$ und $H$, sodass $U = [mm] aHa^{-1}$ [/mm] für ein $a [mm] \in [/mm] G$, also die beiden Untergruppen seien konjugiert - das ganze kannst du dann auf deine zyklisch erzeugten Gruppen anwenden.
Nun hat $U$ die Form [mm] $\{aha^{-1} \mid h \in H \}$ [/mm] also gilt $|U| [mm] \leq [/mm] |H|$.
Nehmen wir an es gelte $|U| < |H|$, das heißt es gibt [mm] $h_1 \neq h_2 \in [/mm] H$ mit [mm] $ah_1a^{-1} [/mm] = [mm] ah_2a^{-1}$. [/mm] Wie kannst du das zu einem Widerspruch führen?

Wenn du das gezeigt hast haben also konjugierte Gruppen immer dieselbe Mächtigkeit.
Das kannst du nun auf die zyklisch erzeugten Gruppen deiner beiden Elemente anwenden und bist fertig.

Du kannst aber wenn du möchtest auch über dein $m [mm] \in \IN$ [/mm] gehen, allerdings nur wenn du weißt, dass ord$(gh)$ bzw. ord$(hg)$ endlich ist.
Sei also ord$(gh)$ endlich, etwa gleich $m [mm] \in \IN$. [/mm]
Schreiben wir [mm] $(gh)^m$ [/mm] und [mm] $(hg)^m$ [/mm] einmal ausmultipliziert hin so wird ersichtlich, dass [mm] $(hg)^m [/mm] = [mm] h(gh)^mh^{-1}$ [/mm] - alternativ kannst du dir das auch klar machen, indem du $hg = [mm] hghh^{-1}$ [/mm] potenzierst, das Prinzip sollte von ähnlichen Matrizen her altbekannt sein.
Ist nun [mm] $(gh)^m [/mm] = e$, so ist also [mm] $(hg)^m [/mm] = [mm] heh^{-1} [/mm] = e$.
Ist also $n$ die Ordnung von $hg$ so weißt du jetzt bereits $n [mm] \mid [/mm] m$ (insbesondere $n$ endlich).
Auf gleiche Weise bekommst du auch $m [mm] \mid [/mm] n$ und damit also $m=n$ wie gewünscht.
Wiegesagt funktioniert dieser Weg nur wenn du endliche Ordnungen voraussetzt, ist $G$ also nicht zwangsläufig endlich würde ich an deiner Stelle über die konjungierten Gruppen gehen.


lg


Schadow

Bezug
                
Bezug
ord(gh) = ord(hg): Frage (beantwortet)
Status: (Frage) beantwortet Status 
Datum: 17:36 Mo 06.08.2012
Autor: ChopSuey

Hi !

vielen Dank für Eure Antworten. Zu dem, was du schriebst Schadow, muss ich mir noch ein paar Gedanken machen. Bin da noch nicht fertig, aber hier blieb ich vor allem hängen:

$ [mm] (gh)^n [/mm] = 1 [mm] \Rightarrow [/mm] h(gh)^ng = hg [mm] \Rightarrow (hg)^n [/mm] = 1 $

die letzte Implikation erschließt sich mir nicht. Wie komm ich zu der Aussage?

Viele Grüße
ChopSuey

Bezug
                        
Bezug
ord(gh) = ord(hg): Antwort
Status: (Antwort) fertig Status 
Datum: 18:20 Mo 06.08.2012
Autor: fred97


> Hi !
>  
> vielen Dank für Eure Antworten. Zu dem, was du schriebst
> Schadow, muss ich mir noch ein paar Gedanken machen. Bin da
> noch nicht fertig, aber hier blieb ich vor allem hängen:
>  
> [mm](gh)^n = 1 \Rightarrow h(gh)^ng = hg \Rightarrow (hg)^n = 1[/mm]
>  
> die letzte Implikation erschließt sich mir nicht. Wie komm
> ich zu der Aussage?

$h(gh)^ng [mm] =(hg)^{n+1}$ [/mm]

Das hatte ich Dir hier

https://matheraum.de/read?i=906114

schon gesagt !

FRED

>  
> Viele Grüße
>  ChopSuey


Bezug
        
Bezug
ord(gh) = ord(hg): Antwort
Status: (Antwort) fertig Status 
Datum: 10:28 Mo 06.08.2012
Autor: fred97

Induktiv sieht man:

              [mm] (hg)^{m+1}=h(gh)^mg. [/mm]

Ist also ord(gh)=n< [mm] \infty, [/mm] so folgt:  [mm] (hg)^{n+1}=hg [/mm] und somit [mm] (hg)^n=e. [/mm]

Damit ist auch ord(hg) endlich und [mm] \le [/mm] n.

Vertauscht man die Rollen von g und h, so sieht man:

              ord(gh) < [mm] \infty \gdw [/mm]  ord(hg) < [mm] \infty. [/mm]

In diesem Fall ist ord(gh)=ord(hg).

FRED

Bezug
Ansicht: [ geschachtelt ] | ^ Forum "Gruppe, Ring, Körper"  | ^^ Alle Foren  | ^ Forenbaum  | Materialien


^ Seitenanfang ^
ev.vorhilfe.de
[ Startseite | Mitglieder | Impressum ]